Prove that f(x)=c has a solution

  • Thread starter Thread starter sergey_le
  • Start date Start date
Click For Summary
To prove that f(x) = c has a solution, it is essential to utilize the continuity of f on the interval (0, ∞). The discussion emphasizes that as x approaches 0 from the right, f tends toward positive infinity. By applying the intermediate value theorem, it can be concluded that for any c greater than 0, there exists an x in (0, ∞) such that f(x) = c. Additionally, demonstrating that the infimum of f over (0, ∞) is 0 supports this conclusion. Overall, the continuity and behavior of f near 0 are crucial in establishing the existence of solutions.
sergey_le
Messages
77
Reaction score
15
Homework Statement
Let function ƒ be f(x)=1/x+sin^2(x) and inff(0,∞)=0.
Prove that every c> 0 has a solution to the equation f(x)=c
Relevant Equations
-
I'm not sure that inff(0,∞)=0 can help But that was the first section of the question so I thought to point it out anyway.
I'm not sure what I'm supposed to do or what I'm supposed to show.
I was thinking of using the right environment of 0 where f aims for infinity but I don't know how it helps me.
 
Physics news on Phys.org
Hint: Show that if ##x \to 0+##, then ##f \to +\infty##. Since ##f## is continuous on ##]0,\infty[##, the intermediate value theorem says that ##f(x)=c## must have a solution for ##c > 0##.

You can do something similar to show that ##\inf f((0, + \infty)) = 0##.
 
  • Like
Likes sergey_le and Greg Bernhardt
Math_QED said:
Hint: Show that if ##x \to 0+##, then ##f \to +\infty##. Since ##f## is continuous on ##]0,\infty[##, the intermediate value theorem says that ##f(x)=c## must have a solution for ##c > 0##.

You can do something similar to show that ##\inf f((0, + \infty)) = 0##.
Thank you.
 
  • Like
Likes member 587159
Question: A clock's minute hand has length 4 and its hour hand has length 3. What is the distance between the tips at the moment when it is increasing most rapidly?(Putnam Exam Question) Answer: Making assumption that both the hands moves at constant angular velocities, the answer is ## \sqrt{7} .## But don't you think this assumption is somewhat doubtful and wrong?

Similar threads

  • · Replies 6 ·
Replies
6
Views
2K
  • · Replies 7 ·
Replies
7
Views
1K
  • · Replies 1 ·
Replies
1
Views
1K
  • · Replies 6 ·
Replies
6
Views
2K
  • · Replies 8 ·
Replies
8
Views
1K
  • · Replies 17 ·
Replies
17
Views
2K
  • · Replies 26 ·
Replies
26
Views
3K
Replies
2
Views
2K
Replies
8
Views
1K
Replies
15
Views
2K